Please confirm topic selection

Are you sure you want to trigger topic in your Anconeus AI algorithm?

Please confirm action

You are done for today with this topic.

Would you like to start learning session with this topic items scheduled for future?

Review Question - QID 214131

In scope icon L 3 A
QID 214131 (Type "214131" in App Search)
Figures A and B are the MRI images of a 51-year-old male who presents with complaints of severe low back pain with radiation in the left posterior thigh for the past 5 days after working out. He denies any numbness, weakness, or bowel/bladder dysfunction. Which treatment modality is indicated at this time for the pathology evident on MRI?
  • A
  • B
  • A
  • B
Attach Treatment Poll
Treatment poll is required to gain more useful feedback from members.
Please enter Question Text
Please enter at least 2 unique options
Please enter at least 2 unique options
Please enter at least 2 unique options